Proving that $arctan(x)+arctan(1/x)=pm pi/2$, could this line of reasoning possibly be correct?











up vote
22
down vote

favorite
11












I know that two questions have already been asked about this exercise, but what I'm asking here is if this solution, which sounds rather strange to me, could possibly be correct. The problems is as follows:




Prove that $,f(x)=arctan(x)+arctan(1/x)= pi/2,$ if $,x>0,$ and $,-pi/2,$ if $,x<0$.




What I did was this: first of all we note that, since $-pi/2<arctan y<pi/2$ for all $y$, $-pi<f(x)<pi$. Now , consider
$$
tanbig(f(x)big)
= tanleft(arctan(x)+arctanleft(frac{1}{x} right)right)
= frac{tanleft(arctan(x)right) + tanleft(arctanleft(frac{1}{x} right)right)}
{1 - tanbig(arctanleft(xright)big) tanleft(arctanleft(frac{1}{x} right)right)}=frac{x+(1/x)}{0},
$$
which is undefined. Since the tangent function is undefined, in $[-pi, pi]$, if and only if its argument is $pm pi/2$, then $f(x)=pm pi/2$. It's easy to see that if $x<0$, then $arctan(x)<0$, hence $f(x)=-pi/2$ and viceversa.



I have found a few other solution to this problem, but I wanted to know if this one is logically acceptable.










share|cite|improve this question




















  • 1




    Thumbs up from my side! Will be curious to see if someone can find a problem with this.
    – Deepak Gupta
    Sep 11 '15 at 6:04






  • 7




    Cute.You can re-phrase it to say "if tan f(x) exists then it equals (x+1/x)/0 ,therefore it doesn't exist,therefore |f(x)|= pi/2"
    – DanielWainfleet
    Sep 11 '15 at 6:28






  • 1




    Reminder to Deepak Gupta .It's infty , not infinity in LaTeX.
    – DanielWainfleet
    Sep 11 '15 at 6:30






  • 4




    Looks good to me. The sequence of equations starting with $tan(f(x))$ assumes (at first) that $tan(f(x))$ is defined, then proves by contradiction that it is not. You could say "assume $tan(f(x))$ is defined" before those equations, but I don't think you really need to. The assumption is implied by the fact that you wrote an equation.
    – David K
    Sep 13 '15 at 16:28






  • 4




    I think there's a problem with the use of the formula related to $tan(x+y)$ because this formula is valid when everything is defined. I think it's better to infere $f(x)=pmfracpi2$ only by showing that $1-tanarctan xtanarctanfrac1x=0$.
    – Mohsen Shahriari
    Sep 13 '15 at 22:36

















up vote
22
down vote

favorite
11












I know that two questions have already been asked about this exercise, but what I'm asking here is if this solution, which sounds rather strange to me, could possibly be correct. The problems is as follows:




Prove that $,f(x)=arctan(x)+arctan(1/x)= pi/2,$ if $,x>0,$ and $,-pi/2,$ if $,x<0$.




What I did was this: first of all we note that, since $-pi/2<arctan y<pi/2$ for all $y$, $-pi<f(x)<pi$. Now , consider
$$
tanbig(f(x)big)
= tanleft(arctan(x)+arctanleft(frac{1}{x} right)right)
= frac{tanleft(arctan(x)right) + tanleft(arctanleft(frac{1}{x} right)right)}
{1 - tanbig(arctanleft(xright)big) tanleft(arctanleft(frac{1}{x} right)right)}=frac{x+(1/x)}{0},
$$
which is undefined. Since the tangent function is undefined, in $[-pi, pi]$, if and only if its argument is $pm pi/2$, then $f(x)=pm pi/2$. It's easy to see that if $x<0$, then $arctan(x)<0$, hence $f(x)=-pi/2$ and viceversa.



I have found a few other solution to this problem, but I wanted to know if this one is logically acceptable.










share|cite|improve this question




















  • 1




    Thumbs up from my side! Will be curious to see if someone can find a problem with this.
    – Deepak Gupta
    Sep 11 '15 at 6:04






  • 7




    Cute.You can re-phrase it to say "if tan f(x) exists then it equals (x+1/x)/0 ,therefore it doesn't exist,therefore |f(x)|= pi/2"
    – DanielWainfleet
    Sep 11 '15 at 6:28






  • 1




    Reminder to Deepak Gupta .It's infty , not infinity in LaTeX.
    – DanielWainfleet
    Sep 11 '15 at 6:30






  • 4




    Looks good to me. The sequence of equations starting with $tan(f(x))$ assumes (at first) that $tan(f(x))$ is defined, then proves by contradiction that it is not. You could say "assume $tan(f(x))$ is defined" before those equations, but I don't think you really need to. The assumption is implied by the fact that you wrote an equation.
    – David K
    Sep 13 '15 at 16:28






  • 4




    I think there's a problem with the use of the formula related to $tan(x+y)$ because this formula is valid when everything is defined. I think it's better to infere $f(x)=pmfracpi2$ only by showing that $1-tanarctan xtanarctanfrac1x=0$.
    – Mohsen Shahriari
    Sep 13 '15 at 22:36















up vote
22
down vote

favorite
11









up vote
22
down vote

favorite
11






11





I know that two questions have already been asked about this exercise, but what I'm asking here is if this solution, which sounds rather strange to me, could possibly be correct. The problems is as follows:




Prove that $,f(x)=arctan(x)+arctan(1/x)= pi/2,$ if $,x>0,$ and $,-pi/2,$ if $,x<0$.




What I did was this: first of all we note that, since $-pi/2<arctan y<pi/2$ for all $y$, $-pi<f(x)<pi$. Now , consider
$$
tanbig(f(x)big)
= tanleft(arctan(x)+arctanleft(frac{1}{x} right)right)
= frac{tanleft(arctan(x)right) + tanleft(arctanleft(frac{1}{x} right)right)}
{1 - tanbig(arctanleft(xright)big) tanleft(arctanleft(frac{1}{x} right)right)}=frac{x+(1/x)}{0},
$$
which is undefined. Since the tangent function is undefined, in $[-pi, pi]$, if and only if its argument is $pm pi/2$, then $f(x)=pm pi/2$. It's easy to see that if $x<0$, then $arctan(x)<0$, hence $f(x)=-pi/2$ and viceversa.



I have found a few other solution to this problem, but I wanted to know if this one is logically acceptable.










share|cite|improve this question















I know that two questions have already been asked about this exercise, but what I'm asking here is if this solution, which sounds rather strange to me, could possibly be correct. The problems is as follows:




Prove that $,f(x)=arctan(x)+arctan(1/x)= pi/2,$ if $,x>0,$ and $,-pi/2,$ if $,x<0$.




What I did was this: first of all we note that, since $-pi/2<arctan y<pi/2$ for all $y$, $-pi<f(x)<pi$. Now , consider
$$
tanbig(f(x)big)
= tanleft(arctan(x)+arctanleft(frac{1}{x} right)right)
= frac{tanleft(arctan(x)right) + tanleft(arctanleft(frac{1}{x} right)right)}
{1 - tanbig(arctanleft(xright)big) tanleft(arctanleft(frac{1}{x} right)right)}=frac{x+(1/x)}{0},
$$
which is undefined. Since the tangent function is undefined, in $[-pi, pi]$, if and only if its argument is $pm pi/2$, then $f(x)=pm pi/2$. It's easy to see that if $x<0$, then $arctan(x)<0$, hence $f(x)=-pi/2$ and viceversa.



I have found a few other solution to this problem, but I wanted to know if this one is logically acceptable.







trigonometry proof-verification






share|cite|improve this question















share|cite|improve this question













share|cite|improve this question




share|cite|improve this question








edited Sep 14 '15 at 3:30









Vlad

4,72932458




4,72932458










asked Sep 11 '15 at 5:59









Nicol

6931716




6931716








  • 1




    Thumbs up from my side! Will be curious to see if someone can find a problem with this.
    – Deepak Gupta
    Sep 11 '15 at 6:04






  • 7




    Cute.You can re-phrase it to say "if tan f(x) exists then it equals (x+1/x)/0 ,therefore it doesn't exist,therefore |f(x)|= pi/2"
    – DanielWainfleet
    Sep 11 '15 at 6:28






  • 1




    Reminder to Deepak Gupta .It's infty , not infinity in LaTeX.
    – DanielWainfleet
    Sep 11 '15 at 6:30






  • 4




    Looks good to me. The sequence of equations starting with $tan(f(x))$ assumes (at first) that $tan(f(x))$ is defined, then proves by contradiction that it is not. You could say "assume $tan(f(x))$ is defined" before those equations, but I don't think you really need to. The assumption is implied by the fact that you wrote an equation.
    – David K
    Sep 13 '15 at 16:28






  • 4




    I think there's a problem with the use of the formula related to $tan(x+y)$ because this formula is valid when everything is defined. I think it's better to infere $f(x)=pmfracpi2$ only by showing that $1-tanarctan xtanarctanfrac1x=0$.
    – Mohsen Shahriari
    Sep 13 '15 at 22:36
















  • 1




    Thumbs up from my side! Will be curious to see if someone can find a problem with this.
    – Deepak Gupta
    Sep 11 '15 at 6:04






  • 7




    Cute.You can re-phrase it to say "if tan f(x) exists then it equals (x+1/x)/0 ,therefore it doesn't exist,therefore |f(x)|= pi/2"
    – DanielWainfleet
    Sep 11 '15 at 6:28






  • 1




    Reminder to Deepak Gupta .It's infty , not infinity in LaTeX.
    – DanielWainfleet
    Sep 11 '15 at 6:30






  • 4




    Looks good to me. The sequence of equations starting with $tan(f(x))$ assumes (at first) that $tan(f(x))$ is defined, then proves by contradiction that it is not. You could say "assume $tan(f(x))$ is defined" before those equations, but I don't think you really need to. The assumption is implied by the fact that you wrote an equation.
    – David K
    Sep 13 '15 at 16:28






  • 4




    I think there's a problem with the use of the formula related to $tan(x+y)$ because this formula is valid when everything is defined. I think it's better to infere $f(x)=pmfracpi2$ only by showing that $1-tanarctan xtanarctanfrac1x=0$.
    – Mohsen Shahriari
    Sep 13 '15 at 22:36










1




1




Thumbs up from my side! Will be curious to see if someone can find a problem with this.
– Deepak Gupta
Sep 11 '15 at 6:04




Thumbs up from my side! Will be curious to see if someone can find a problem with this.
– Deepak Gupta
Sep 11 '15 at 6:04




7




7




Cute.You can re-phrase it to say "if tan f(x) exists then it equals (x+1/x)/0 ,therefore it doesn't exist,therefore |f(x)|= pi/2"
– DanielWainfleet
Sep 11 '15 at 6:28




Cute.You can re-phrase it to say "if tan f(x) exists then it equals (x+1/x)/0 ,therefore it doesn't exist,therefore |f(x)|= pi/2"
– DanielWainfleet
Sep 11 '15 at 6:28




1




1




Reminder to Deepak Gupta .It's infty , not infinity in LaTeX.
– DanielWainfleet
Sep 11 '15 at 6:30




Reminder to Deepak Gupta .It's infty , not infinity in LaTeX.
– DanielWainfleet
Sep 11 '15 at 6:30




4




4




Looks good to me. The sequence of equations starting with $tan(f(x))$ assumes (at first) that $tan(f(x))$ is defined, then proves by contradiction that it is not. You could say "assume $tan(f(x))$ is defined" before those equations, but I don't think you really need to. The assumption is implied by the fact that you wrote an equation.
– David K
Sep 13 '15 at 16:28




Looks good to me. The sequence of equations starting with $tan(f(x))$ assumes (at first) that $tan(f(x))$ is defined, then proves by contradiction that it is not. You could say "assume $tan(f(x))$ is defined" before those equations, but I don't think you really need to. The assumption is implied by the fact that you wrote an equation.
– David K
Sep 13 '15 at 16:28




4




4




I think there's a problem with the use of the formula related to $tan(x+y)$ because this formula is valid when everything is defined. I think it's better to infere $f(x)=pmfracpi2$ only by showing that $1-tanarctan xtanarctanfrac1x=0$.
– Mohsen Shahriari
Sep 13 '15 at 22:36






I think there's a problem with the use of the formula related to $tan(x+y)$ because this formula is valid when everything is defined. I think it's better to infere $f(x)=pmfracpi2$ only by showing that $1-tanarctan xtanarctanfrac1x=0$.
– Mohsen Shahriari
Sep 13 '15 at 22:36












8 Answers
8






active

oldest

votes

















up vote
3
down vote



accepted
+50










Whatever you have done is correct and nothing is wrong there.You could get the same as follows also:
Note that : $tan^{-1}x=tan^{-1}1/x -pi$, when $xlt0$...(A) and is equal to $tan^{-1}1/x$ when $xgt0$..(B)



PROOF:I am proving for $x$ being negative.For +ve $x$ it's quite easy to prove.



Suppose, $y=-z$, ($z$ is positive ), hen $cot^{-1}(y)=cot^{-1}(-z)=pi -cot^{-1}(z)$ Now you might be knowing :$cot^{-1}(z)=tan^{-1}(1/z)$ for positive $z$, hence $cot^{-1}(-z)=pi - tan^{-1}(1/z)$. Now substitue $z=-y$, then use $tan^{-1}(-1/y)=-tan^{-1}(1/y)$ and you get the result.



Now you may argue: why $cot^{-1}(-z)=pi-cot^{-1}z$?



Well, it's because :Let $z=cottheta$, $0<theta<pi$[principal branch of $cot$]. So $-z=cot(pi-theta)$ , Note that $pi- theta$ is also in between $0$ and $pi$. So you can define $cot^{-1}(-z)$, which in this case will be: $pi -theta$, Now put $theta=cot^{-1}(z)$, Hence proved.



So coming to: $tan^{-1}x +tan^{-1}1/x=I$, say



So $I=tan^{-1}x +cot^{-1}x=pi/2$, when $xgt0$ [by (B)]



And $I=tan^{-1}x +cot^{-1}x-pi=pi/2-pi=-pi/2$, when $x lt0$ [by (A)]






share|cite|improve this answer



















  • 1




    Do you see a typographical difference between $tan^{-1}x$=$tan^{-1}1/x$ -$pi$ and $tan^{-1}x=tan^{-1}1/x -pi$? The latter is proper usage. ${}qquad{}$
    – Michael Hardy
    Sep 19 '15 at 0:48










  • @MichaelHardy,both are the same.What do you actually want to say?Please explain.
    – Suraj_Singh
    Sep 19 '15 at 1:34












  • @MichaelHardy,oh! now I see.Thanks for information!
    – Suraj_Singh
    Sep 19 '15 at 1:38










  • @MichaelHardy,anyways what is wrong with the first usage??Please explain.
    – Suraj_Singh
    Sep 19 '15 at 1:39






  • 1




    Look at it. In the first example, the minus sign before $pi$ looks like a hypen rather than a minus sign and lacks spacing to its left and right, thus $1/xtext{-}pi$ instead of $1/x-pi$. The "equals" sign is also to small for the font size and lacks spacing no the left and right, thus: $tan^{-1}x$=$tan^{-1}$ etc. instead of $tan^{-1}x=tan$ etc. ${}qquad{}$
    – Michael Hardy
    Sep 19 '15 at 2:05




















up vote
5
down vote













Right triangle with legs $1$ and $x$, just remembering the sum of all the angles of a triangle, is $pi$. In case of a right triangle, $alpha+beta={piover2}$ for any $x$.



enter image description here






share|cite|improve this answer






























    up vote
    3
    down vote













    Here's a proof
    in the case when
    $x > 0$.



    Draw a right triangle
    with legs $1$ and $x$,
    with the leg of length $x$
    opposite angle $A$.



    Then
    $tan(A) = x$
    and
    $tan(B) = 1/x$,
    so $A = arctan(x)$
    and
    $B = arctan(1/x)$.



    Since $A+B = pi/2$,
    $arctan(x)+arctan(1/x)
    = pi/2
    $.






    share|cite|improve this answer




























      up vote
      2
      down vote













      The tangent function should be defined to take the value $infty$ at $pmpi/2$, and this $infty$ is neither $+infty$ or $-infty$, but is the $infty$ that is approached by going either in the positive direction or the negative direction. That makes the tangent function everywhere continuous, including continuity at $pmpi/2$.



      If one also identifies $+pi/2$ with $-pi/2$ so that the domain of the tangent function is topologically a circle one of whose points is $+pi/2=-pi/2$, then the tangent function is one-to-one. There is only one point in its domain that maps to $infty$, namely $+pi/2=-pi/2$.



      After that, there remains the question of whether the standard identity for the tangent of a sum applies when $infty$ occurs among the values of the functions involved. To address that we should also take $infty$ (not $+infty$ and not $-infty$) to be the value of a rational function wherever it has a vertical asymptote. This makes rational functions everywhere continuous. Then we have
      begin{align}
      tan(alpha+beta) & overset{hugetext{?}}= frac{sin(alpha+beta)}{cos(alpha+beta)} tag 1 \[10pt]
      & {} = frac{sinalphacosbeta+cosalphasinbeta}{cosalphacosbeta-sinalphasinbeta}. tag 2
      end{align}
      Notice that in $(1)$, the sine and cosine cannot both be $0$, so we need not consider what happens then.



      In the standard argument, we divide both the numerator and the denominator by $cosalphacosbeta$, getting this:
      $$
      frac{sinalphacosbeta+cosalphasinbeta}{cosalphacosbeta-sinalphasinbeta} = frac{frac{sinalpha}{cosalpha} - frac{sinbeta}{cosbeta}}{1 - frac{sinalpha}{cosalpha}cdotfrac{sinbeta}{cosbeta}} tag 3
      $$
      All this is valid when there are no $0$s in denominators. We need to deal with $0$s in denominators.



      First consider the case in which the denominator in $(1)$ is $0$ but $cosalphane0necosbeta$. Then the denominator on the right side of $(3)$ is $0$. But the numerator on the right is not $0$, since the numerator on the left is not $0$; the numerator on the left side of $(3)$ cannot be $0$ since $sin(alpha+beta) ne 0$. Hence the standard identity holds in that case.



      Next consider the case where $cos(alpha+beta)=0$ and $cosalpha=0$. Then



      $$
      fracpi2 = - fracpi2 = alpha+beta = fracpi2+beta = -fracpi2 + beta
      $$
      and we're done.






      share|cite|improve this answer




























        up vote
        2
        down vote













        You can't compute $tantheta$ when $theta=pmfracpi2$, but you can compute $costheta$ instead.
        For $a,bin mathbb{R}$ with $abne 1$ we have
        begin{eqnarray}
        cos^2(arctan(a)+arctan(b))&=&frac{1}{1+tan^2(arctan(a)+arctan(b))}=frac{1}{1+left(frac{a+b}{1-ab}right)^2}\
        &=&frac{(1-ab)^2}{(1-ab)^2+(ab)^2}.
        end{eqnarray}
        It follows that
        $$
        cos^2(f(x))=lim_{ato x,bto x^{-1}}cos^2(arctan(a)+arctan(b))=lim_{ato x,bto x^{-1}}frac{(1-ab)^2}{(1-ab)^2+(ab)^2}=0,
        $$
        i.e. $f(x)=pmfracpi2$. Since $f$ is continuous on $(-infty,0)$ and on $(0,infty)$, we deduce that
        $$
        f(x)=f(-1)=2arctan(-1)=-fracpi2 quad forall x<0,
        $$
        and
        $$
        f(x)=-f(-x)=fracpi2 quad forall x>0
        $$





        Added
        The function $f: xmapsto f(x)=arctan(x)+arctan(x^{-1})$ defined and differentiable of $mathbb{R}setminus{0}$. For every $xne 0$ we have
        $$
        f'(x)=dfrac{1}{1+x^2}+dfrac{-x^{-2}}{1+x^{-2}}=dfrac{1}{1+x^2}-dfrac{1}{x^2+1}=0
        $$
        Therefore, $f$ is constant on each connected component of $mathbb{R}setminus{0}$.
        Since
        $$
        f(1)=2arctan(1)=2dfrac{pi}{4}=dfrac{pi}{2},quad f(-1)=-f(1)=-dfrac{pi}{2},
        $$
        it follows that
        $$
        arctan(x)+arctanleft(dfrac{1}{x}right)=begin{cases}-dfrac{pi}{2} &text{ if } x<0\
        dfrac{pi}{2} &text{ if } x>0
        end{cases}.
        $$






        share|cite|improve this answer






























          up vote
          1
          down vote













          I just figured I'd throw this in.



          Let $f(x) = arctan(x) + arctan(1/x)$ for all $x in (0, infty)$.



          Then $f'(x) = dfrac{1}{1+x^2} - dfrac{dfrac{1}{x^2}}{1 + dfrac{1}{x^2}} = 0$.



          Hence $f(x)$ is constant on $(0, infty)$.



          Since $f(1) = dfrac{pi}{4} + dfrac{pi}{4} = dfrac{pi}{2}$,
          we conclude that



          $f(x) = dfrac{pi}{2}$ for all $x in (0, infty)$.





          Addendum



          If you're not ready for calculus, for the same
          $x in (0, infty)$,
          Consider the point
          $P = (1, x)$, in the first quadrant, with corresponding angle
          $0 lt theta lt dfrac{pi}{2}$.



          Let $hat{theta} = dfrac{pi}{2} - theta$.
          Then, also, $0 lt hat{theta} lt dfrac{pi}{2}$ and
          $tan(hat theta)
          = tan left( dfrac{pi}{2} - theta right)
          = cot theta = dfrac 1x$



          It follows that
          $arctan x + arctan dfrac 1x
          = theta + hat theta
          = dfrac{pi}{2}$





          For all $x in (-infty, 0)$, we have



          $arctan x + arctan dfrac 1x =
          -left(arctan(-x) + arctan left(-dfrac 1x right) right) =
          -f(-x) = -dfrac{pi}{2}$.






          share|cite|improve this answer






























            up vote
            0
            down vote













            Your logic is entirely correct.



            The functions $tan^{-1} x $ and $tan^{-1} (1/x ) $ are both odd functions. Accordingly their additive $tan^{-1} x + tan^{-1} (1/x ) $ must be odd with $ + pi/2 $ value for positive arguments and $ - pi/2 $ value for negative arguments.



            This function results by subtracting $ pi/2$ from the step function 0 to $pi$ jump at x = 0 and so making it an odd function of opposite sign for arguments of opposite sign, defining the $ f(x) $ to be $pi/2 $ for $ x>0, 0$ for $ x=0 $ and $ - pi/2 $ for $ x<0. $






            share|cite|improve this answer






























              up vote
              0
              down vote













              An alternative proof using calculus.



              Let $f(x) = arctan(x) + arctan(1/x)$ for all real $x$ except $0$.



              $$frac {df(x)} {dx}= frac{1}{x^2+1} + frac {1}{1+1/x^2}*frac{-1}{x^2} = frac{1}{x^2+1} - frac{1}{x^2+1} = 0 $$



              Integrating on both sides we get



              $$f(x) = arctan(x) + arctan(1/x) = C,$$ where $C$ is a constant.



              Since the function is discontinuous at $x=0$ the constant value can be different on both sides.



              It can be obtained for $x>0$ by for example setting $x=1$:
              $$f(1) = arctan(1) + arctan(1) = pi/4 + pi/4 = pi/2 $$
              Similarly, for $x=-1$:
              $$f(1) = arctan(-1) + arctan(-1) = -pi/4 -pi/4 = -pi/2 $$



              So, in conclusion, $$arctan(x) + arctan(1/x) = pi/2 $$ for $x>0$
              and
              $$arctan(x) + arctan(1/x) = -pi/2 $$ for $x<0$.






              share|cite|improve this answer























                Your Answer





                StackExchange.ifUsing("editor", function () {
                return StackExchange.using("mathjaxEditing", function () {
                StackExchange.MarkdownEditor.creationCallbacks.add(function (editor, postfix) {
                StackExchange.mathjaxEditing.prepareWmdForMathJax(editor, postfix, [["$", "$"], ["\\(","\\)"]]);
                });
                });
                }, "mathjax-editing");

                StackExchange.ready(function() {
                var channelOptions = {
                tags: "".split(" "),
                id: "69"
                };
                initTagRenderer("".split(" "), "".split(" "), channelOptions);

                StackExchange.using("externalEditor", function() {
                // Have to fire editor after snippets, if snippets enabled
                if (StackExchange.settings.snippets.snippetsEnabled) {
                StackExchange.using("snippets", function() {
                createEditor();
                });
                }
                else {
                createEditor();
                }
                });

                function createEditor() {
                StackExchange.prepareEditor({
                heartbeatType: 'answer',
                convertImagesToLinks: true,
                noModals: true,
                showLowRepImageUploadWarning: true,
                reputationToPostImages: 10,
                bindNavPrevention: true,
                postfix: "",
                imageUploader: {
                brandingHtml: "Powered by u003ca class="icon-imgur-white" href="https://imgur.com/"u003eu003c/au003e",
                contentPolicyHtml: "User contributions licensed under u003ca href="https://creativecommons.org/licenses/by-sa/3.0/"u003ecc by-sa 3.0 with attribution requiredu003c/au003e u003ca href="https://stackoverflow.com/legal/content-policy"u003e(content policy)u003c/au003e",
                allowUrls: true
                },
                noCode: true, onDemand: true,
                discardSelector: ".discard-answer"
                ,immediatelyShowMarkdownHelp:true
                });


                }
                });














                 

                draft saved


                draft discarded


















                StackExchange.ready(
                function () {
                StackExchange.openid.initPostLogin('.new-post-login', 'https%3a%2f%2fmath.stackexchange.com%2fquestions%2f1430520%2fproving-that-arctanx-arctan1-x-pm-pi-2-could-this-line-of-reasoning-p%23new-answer', 'question_page');
                }
                );

                Post as a guest















                Required, but never shown

























                8 Answers
                8






                active

                oldest

                votes








                8 Answers
                8






                active

                oldest

                votes









                active

                oldest

                votes






                active

                oldest

                votes








                up vote
                3
                down vote



                accepted
                +50










                Whatever you have done is correct and nothing is wrong there.You could get the same as follows also:
                Note that : $tan^{-1}x=tan^{-1}1/x -pi$, when $xlt0$...(A) and is equal to $tan^{-1}1/x$ when $xgt0$..(B)



                PROOF:I am proving for $x$ being negative.For +ve $x$ it's quite easy to prove.



                Suppose, $y=-z$, ($z$ is positive ), hen $cot^{-1}(y)=cot^{-1}(-z)=pi -cot^{-1}(z)$ Now you might be knowing :$cot^{-1}(z)=tan^{-1}(1/z)$ for positive $z$, hence $cot^{-1}(-z)=pi - tan^{-1}(1/z)$. Now substitue $z=-y$, then use $tan^{-1}(-1/y)=-tan^{-1}(1/y)$ and you get the result.



                Now you may argue: why $cot^{-1}(-z)=pi-cot^{-1}z$?



                Well, it's because :Let $z=cottheta$, $0<theta<pi$[principal branch of $cot$]. So $-z=cot(pi-theta)$ , Note that $pi- theta$ is also in between $0$ and $pi$. So you can define $cot^{-1}(-z)$, which in this case will be: $pi -theta$, Now put $theta=cot^{-1}(z)$, Hence proved.



                So coming to: $tan^{-1}x +tan^{-1}1/x=I$, say



                So $I=tan^{-1}x +cot^{-1}x=pi/2$, when $xgt0$ [by (B)]



                And $I=tan^{-1}x +cot^{-1}x-pi=pi/2-pi=-pi/2$, when $x lt0$ [by (A)]






                share|cite|improve this answer



















                • 1




                  Do you see a typographical difference between $tan^{-1}x$=$tan^{-1}1/x$ -$pi$ and $tan^{-1}x=tan^{-1}1/x -pi$? The latter is proper usage. ${}qquad{}$
                  – Michael Hardy
                  Sep 19 '15 at 0:48










                • @MichaelHardy,both are the same.What do you actually want to say?Please explain.
                  – Suraj_Singh
                  Sep 19 '15 at 1:34












                • @MichaelHardy,oh! now I see.Thanks for information!
                  – Suraj_Singh
                  Sep 19 '15 at 1:38










                • @MichaelHardy,anyways what is wrong with the first usage??Please explain.
                  – Suraj_Singh
                  Sep 19 '15 at 1:39






                • 1




                  Look at it. In the first example, the minus sign before $pi$ looks like a hypen rather than a minus sign and lacks spacing to its left and right, thus $1/xtext{-}pi$ instead of $1/x-pi$. The "equals" sign is also to small for the font size and lacks spacing no the left and right, thus: $tan^{-1}x$=$tan^{-1}$ etc. instead of $tan^{-1}x=tan$ etc. ${}qquad{}$
                  – Michael Hardy
                  Sep 19 '15 at 2:05

















                up vote
                3
                down vote



                accepted
                +50










                Whatever you have done is correct and nothing is wrong there.You could get the same as follows also:
                Note that : $tan^{-1}x=tan^{-1}1/x -pi$, when $xlt0$...(A) and is equal to $tan^{-1}1/x$ when $xgt0$..(B)



                PROOF:I am proving for $x$ being negative.For +ve $x$ it's quite easy to prove.



                Suppose, $y=-z$, ($z$ is positive ), hen $cot^{-1}(y)=cot^{-1}(-z)=pi -cot^{-1}(z)$ Now you might be knowing :$cot^{-1}(z)=tan^{-1}(1/z)$ for positive $z$, hence $cot^{-1}(-z)=pi - tan^{-1}(1/z)$. Now substitue $z=-y$, then use $tan^{-1}(-1/y)=-tan^{-1}(1/y)$ and you get the result.



                Now you may argue: why $cot^{-1}(-z)=pi-cot^{-1}z$?



                Well, it's because :Let $z=cottheta$, $0<theta<pi$[principal branch of $cot$]. So $-z=cot(pi-theta)$ , Note that $pi- theta$ is also in between $0$ and $pi$. So you can define $cot^{-1}(-z)$, which in this case will be: $pi -theta$, Now put $theta=cot^{-1}(z)$, Hence proved.



                So coming to: $tan^{-1}x +tan^{-1}1/x=I$, say



                So $I=tan^{-1}x +cot^{-1}x=pi/2$, when $xgt0$ [by (B)]



                And $I=tan^{-1}x +cot^{-1}x-pi=pi/2-pi=-pi/2$, when $x lt0$ [by (A)]






                share|cite|improve this answer



















                • 1




                  Do you see a typographical difference between $tan^{-1}x$=$tan^{-1}1/x$ -$pi$ and $tan^{-1}x=tan^{-1}1/x -pi$? The latter is proper usage. ${}qquad{}$
                  – Michael Hardy
                  Sep 19 '15 at 0:48










                • @MichaelHardy,both are the same.What do you actually want to say?Please explain.
                  – Suraj_Singh
                  Sep 19 '15 at 1:34












                • @MichaelHardy,oh! now I see.Thanks for information!
                  – Suraj_Singh
                  Sep 19 '15 at 1:38










                • @MichaelHardy,anyways what is wrong with the first usage??Please explain.
                  – Suraj_Singh
                  Sep 19 '15 at 1:39






                • 1




                  Look at it. In the first example, the minus sign before $pi$ looks like a hypen rather than a minus sign and lacks spacing to its left and right, thus $1/xtext{-}pi$ instead of $1/x-pi$. The "equals" sign is also to small for the font size and lacks spacing no the left and right, thus: $tan^{-1}x$=$tan^{-1}$ etc. instead of $tan^{-1}x=tan$ etc. ${}qquad{}$
                  – Michael Hardy
                  Sep 19 '15 at 2:05















                up vote
                3
                down vote



                accepted
                +50







                up vote
                3
                down vote



                accepted
                +50




                +50




                Whatever you have done is correct and nothing is wrong there.You could get the same as follows also:
                Note that : $tan^{-1}x=tan^{-1}1/x -pi$, when $xlt0$...(A) and is equal to $tan^{-1}1/x$ when $xgt0$..(B)



                PROOF:I am proving for $x$ being negative.For +ve $x$ it's quite easy to prove.



                Suppose, $y=-z$, ($z$ is positive ), hen $cot^{-1}(y)=cot^{-1}(-z)=pi -cot^{-1}(z)$ Now you might be knowing :$cot^{-1}(z)=tan^{-1}(1/z)$ for positive $z$, hence $cot^{-1}(-z)=pi - tan^{-1}(1/z)$. Now substitue $z=-y$, then use $tan^{-1}(-1/y)=-tan^{-1}(1/y)$ and you get the result.



                Now you may argue: why $cot^{-1}(-z)=pi-cot^{-1}z$?



                Well, it's because :Let $z=cottheta$, $0<theta<pi$[principal branch of $cot$]. So $-z=cot(pi-theta)$ , Note that $pi- theta$ is also in between $0$ and $pi$. So you can define $cot^{-1}(-z)$, which in this case will be: $pi -theta$, Now put $theta=cot^{-1}(z)$, Hence proved.



                So coming to: $tan^{-1}x +tan^{-1}1/x=I$, say



                So $I=tan^{-1}x +cot^{-1}x=pi/2$, when $xgt0$ [by (B)]



                And $I=tan^{-1}x +cot^{-1}x-pi=pi/2-pi=-pi/2$, when $x lt0$ [by (A)]






                share|cite|improve this answer














                Whatever you have done is correct and nothing is wrong there.You could get the same as follows also:
                Note that : $tan^{-1}x=tan^{-1}1/x -pi$, when $xlt0$...(A) and is equal to $tan^{-1}1/x$ when $xgt0$..(B)



                PROOF:I am proving for $x$ being negative.For +ve $x$ it's quite easy to prove.



                Suppose, $y=-z$, ($z$ is positive ), hen $cot^{-1}(y)=cot^{-1}(-z)=pi -cot^{-1}(z)$ Now you might be knowing :$cot^{-1}(z)=tan^{-1}(1/z)$ for positive $z$, hence $cot^{-1}(-z)=pi - tan^{-1}(1/z)$. Now substitue $z=-y$, then use $tan^{-1}(-1/y)=-tan^{-1}(1/y)$ and you get the result.



                Now you may argue: why $cot^{-1}(-z)=pi-cot^{-1}z$?



                Well, it's because :Let $z=cottheta$, $0<theta<pi$[principal branch of $cot$]. So $-z=cot(pi-theta)$ , Note that $pi- theta$ is also in between $0$ and $pi$. So you can define $cot^{-1}(-z)$, which in this case will be: $pi -theta$, Now put $theta=cot^{-1}(z)$, Hence proved.



                So coming to: $tan^{-1}x +tan^{-1}1/x=I$, say



                So $I=tan^{-1}x +cot^{-1}x=pi/2$, when $xgt0$ [by (B)]



                And $I=tan^{-1}x +cot^{-1}x-pi=pi/2-pi=-pi/2$, when $x lt0$ [by (A)]







                share|cite|improve this answer














                share|cite|improve this answer



                share|cite|improve this answer








                edited Sep 19 '15 at 0:51









                Michael Hardy

                1




                1










                answered Sep 13 '15 at 15:46









                Suraj_Singh

                765516




                765516








                • 1




                  Do you see a typographical difference between $tan^{-1}x$=$tan^{-1}1/x$ -$pi$ and $tan^{-1}x=tan^{-1}1/x -pi$? The latter is proper usage. ${}qquad{}$
                  – Michael Hardy
                  Sep 19 '15 at 0:48










                • @MichaelHardy,both are the same.What do you actually want to say?Please explain.
                  – Suraj_Singh
                  Sep 19 '15 at 1:34












                • @MichaelHardy,oh! now I see.Thanks for information!
                  – Suraj_Singh
                  Sep 19 '15 at 1:38










                • @MichaelHardy,anyways what is wrong with the first usage??Please explain.
                  – Suraj_Singh
                  Sep 19 '15 at 1:39






                • 1




                  Look at it. In the first example, the minus sign before $pi$ looks like a hypen rather than a minus sign and lacks spacing to its left and right, thus $1/xtext{-}pi$ instead of $1/x-pi$. The "equals" sign is also to small for the font size and lacks spacing no the left and right, thus: $tan^{-1}x$=$tan^{-1}$ etc. instead of $tan^{-1}x=tan$ etc. ${}qquad{}$
                  – Michael Hardy
                  Sep 19 '15 at 2:05
















                • 1




                  Do you see a typographical difference between $tan^{-1}x$=$tan^{-1}1/x$ -$pi$ and $tan^{-1}x=tan^{-1}1/x -pi$? The latter is proper usage. ${}qquad{}$
                  – Michael Hardy
                  Sep 19 '15 at 0:48










                • @MichaelHardy,both are the same.What do you actually want to say?Please explain.
                  – Suraj_Singh
                  Sep 19 '15 at 1:34












                • @MichaelHardy,oh! now I see.Thanks for information!
                  – Suraj_Singh
                  Sep 19 '15 at 1:38










                • @MichaelHardy,anyways what is wrong with the first usage??Please explain.
                  – Suraj_Singh
                  Sep 19 '15 at 1:39






                • 1




                  Look at it. In the first example, the minus sign before $pi$ looks like a hypen rather than a minus sign and lacks spacing to its left and right, thus $1/xtext{-}pi$ instead of $1/x-pi$. The "equals" sign is also to small for the font size and lacks spacing no the left and right, thus: $tan^{-1}x$=$tan^{-1}$ etc. instead of $tan^{-1}x=tan$ etc. ${}qquad{}$
                  – Michael Hardy
                  Sep 19 '15 at 2:05










                1




                1




                Do you see a typographical difference between $tan^{-1}x$=$tan^{-1}1/x$ -$pi$ and $tan^{-1}x=tan^{-1}1/x -pi$? The latter is proper usage. ${}qquad{}$
                – Michael Hardy
                Sep 19 '15 at 0:48




                Do you see a typographical difference between $tan^{-1}x$=$tan^{-1}1/x$ -$pi$ and $tan^{-1}x=tan^{-1}1/x -pi$? The latter is proper usage. ${}qquad{}$
                – Michael Hardy
                Sep 19 '15 at 0:48












                @MichaelHardy,both are the same.What do you actually want to say?Please explain.
                – Suraj_Singh
                Sep 19 '15 at 1:34






                @MichaelHardy,both are the same.What do you actually want to say?Please explain.
                – Suraj_Singh
                Sep 19 '15 at 1:34














                @MichaelHardy,oh! now I see.Thanks for information!
                – Suraj_Singh
                Sep 19 '15 at 1:38




                @MichaelHardy,oh! now I see.Thanks for information!
                – Suraj_Singh
                Sep 19 '15 at 1:38












                @MichaelHardy,anyways what is wrong with the first usage??Please explain.
                – Suraj_Singh
                Sep 19 '15 at 1:39




                @MichaelHardy,anyways what is wrong with the first usage??Please explain.
                – Suraj_Singh
                Sep 19 '15 at 1:39




                1




                1




                Look at it. In the first example, the minus sign before $pi$ looks like a hypen rather than a minus sign and lacks spacing to its left and right, thus $1/xtext{-}pi$ instead of $1/x-pi$. The "equals" sign is also to small for the font size and lacks spacing no the left and right, thus: $tan^{-1}x$=$tan^{-1}$ etc. instead of $tan^{-1}x=tan$ etc. ${}qquad{}$
                – Michael Hardy
                Sep 19 '15 at 2:05






                Look at it. In the first example, the minus sign before $pi$ looks like a hypen rather than a minus sign and lacks spacing to its left and right, thus $1/xtext{-}pi$ instead of $1/x-pi$. The "equals" sign is also to small for the font size and lacks spacing no the left and right, thus: $tan^{-1}x$=$tan^{-1}$ etc. instead of $tan^{-1}x=tan$ etc. ${}qquad{}$
                – Michael Hardy
                Sep 19 '15 at 2:05












                up vote
                5
                down vote













                Right triangle with legs $1$ and $x$, just remembering the sum of all the angles of a triangle, is $pi$. In case of a right triangle, $alpha+beta={piover2}$ for any $x$.



                enter image description here






                share|cite|improve this answer



























                  up vote
                  5
                  down vote













                  Right triangle with legs $1$ and $x$, just remembering the sum of all the angles of a triangle, is $pi$. In case of a right triangle, $alpha+beta={piover2}$ for any $x$.



                  enter image description here






                  share|cite|improve this answer

























                    up vote
                    5
                    down vote










                    up vote
                    5
                    down vote









                    Right triangle with legs $1$ and $x$, just remembering the sum of all the angles of a triangle, is $pi$. In case of a right triangle, $alpha+beta={piover2}$ for any $x$.



                    enter image description here






                    share|cite|improve this answer














                    Right triangle with legs $1$ and $x$, just remembering the sum of all the angles of a triangle, is $pi$. In case of a right triangle, $alpha+beta={piover2}$ for any $x$.



                    enter image description here







                    share|cite|improve this answer














                    share|cite|improve this answer



                    share|cite|improve this answer








                    edited Feb 16 '17 at 21:13









                    Daniel Fischer

                    172k16158281




                    172k16158281










                    answered Feb 16 '17 at 19:26









                    Gustavo Mezzovilla

                    30737




                    30737






















                        up vote
                        3
                        down vote













                        Here's a proof
                        in the case when
                        $x > 0$.



                        Draw a right triangle
                        with legs $1$ and $x$,
                        with the leg of length $x$
                        opposite angle $A$.



                        Then
                        $tan(A) = x$
                        and
                        $tan(B) = 1/x$,
                        so $A = arctan(x)$
                        and
                        $B = arctan(1/x)$.



                        Since $A+B = pi/2$,
                        $arctan(x)+arctan(1/x)
                        = pi/2
                        $.






                        share|cite|improve this answer

























                          up vote
                          3
                          down vote













                          Here's a proof
                          in the case when
                          $x > 0$.



                          Draw a right triangle
                          with legs $1$ and $x$,
                          with the leg of length $x$
                          opposite angle $A$.



                          Then
                          $tan(A) = x$
                          and
                          $tan(B) = 1/x$,
                          so $A = arctan(x)$
                          and
                          $B = arctan(1/x)$.



                          Since $A+B = pi/2$,
                          $arctan(x)+arctan(1/x)
                          = pi/2
                          $.






                          share|cite|improve this answer























                            up vote
                            3
                            down vote










                            up vote
                            3
                            down vote









                            Here's a proof
                            in the case when
                            $x > 0$.



                            Draw a right triangle
                            with legs $1$ and $x$,
                            with the leg of length $x$
                            opposite angle $A$.



                            Then
                            $tan(A) = x$
                            and
                            $tan(B) = 1/x$,
                            so $A = arctan(x)$
                            and
                            $B = arctan(1/x)$.



                            Since $A+B = pi/2$,
                            $arctan(x)+arctan(1/x)
                            = pi/2
                            $.






                            share|cite|improve this answer












                            Here's a proof
                            in the case when
                            $x > 0$.



                            Draw a right triangle
                            with legs $1$ and $x$,
                            with the leg of length $x$
                            opposite angle $A$.



                            Then
                            $tan(A) = x$
                            and
                            $tan(B) = 1/x$,
                            so $A = arctan(x)$
                            and
                            $B = arctan(1/x)$.



                            Since $A+B = pi/2$,
                            $arctan(x)+arctan(1/x)
                            = pi/2
                            $.







                            share|cite|improve this answer












                            share|cite|improve this answer



                            share|cite|improve this answer










                            answered Sep 19 '15 at 0:23









                            marty cohen

                            71.4k546123




                            71.4k546123






















                                up vote
                                2
                                down vote













                                The tangent function should be defined to take the value $infty$ at $pmpi/2$, and this $infty$ is neither $+infty$ or $-infty$, but is the $infty$ that is approached by going either in the positive direction or the negative direction. That makes the tangent function everywhere continuous, including continuity at $pmpi/2$.



                                If one also identifies $+pi/2$ with $-pi/2$ so that the domain of the tangent function is topologically a circle one of whose points is $+pi/2=-pi/2$, then the tangent function is one-to-one. There is only one point in its domain that maps to $infty$, namely $+pi/2=-pi/2$.



                                After that, there remains the question of whether the standard identity for the tangent of a sum applies when $infty$ occurs among the values of the functions involved. To address that we should also take $infty$ (not $+infty$ and not $-infty$) to be the value of a rational function wherever it has a vertical asymptote. This makes rational functions everywhere continuous. Then we have
                                begin{align}
                                tan(alpha+beta) & overset{hugetext{?}}= frac{sin(alpha+beta)}{cos(alpha+beta)} tag 1 \[10pt]
                                & {} = frac{sinalphacosbeta+cosalphasinbeta}{cosalphacosbeta-sinalphasinbeta}. tag 2
                                end{align}
                                Notice that in $(1)$, the sine and cosine cannot both be $0$, so we need not consider what happens then.



                                In the standard argument, we divide both the numerator and the denominator by $cosalphacosbeta$, getting this:
                                $$
                                frac{sinalphacosbeta+cosalphasinbeta}{cosalphacosbeta-sinalphasinbeta} = frac{frac{sinalpha}{cosalpha} - frac{sinbeta}{cosbeta}}{1 - frac{sinalpha}{cosalpha}cdotfrac{sinbeta}{cosbeta}} tag 3
                                $$
                                All this is valid when there are no $0$s in denominators. We need to deal with $0$s in denominators.



                                First consider the case in which the denominator in $(1)$ is $0$ but $cosalphane0necosbeta$. Then the denominator on the right side of $(3)$ is $0$. But the numerator on the right is not $0$, since the numerator on the left is not $0$; the numerator on the left side of $(3)$ cannot be $0$ since $sin(alpha+beta) ne 0$. Hence the standard identity holds in that case.



                                Next consider the case where $cos(alpha+beta)=0$ and $cosalpha=0$. Then



                                $$
                                fracpi2 = - fracpi2 = alpha+beta = fracpi2+beta = -fracpi2 + beta
                                $$
                                and we're done.






                                share|cite|improve this answer

























                                  up vote
                                  2
                                  down vote













                                  The tangent function should be defined to take the value $infty$ at $pmpi/2$, and this $infty$ is neither $+infty$ or $-infty$, but is the $infty$ that is approached by going either in the positive direction or the negative direction. That makes the tangent function everywhere continuous, including continuity at $pmpi/2$.



                                  If one also identifies $+pi/2$ with $-pi/2$ so that the domain of the tangent function is topologically a circle one of whose points is $+pi/2=-pi/2$, then the tangent function is one-to-one. There is only one point in its domain that maps to $infty$, namely $+pi/2=-pi/2$.



                                  After that, there remains the question of whether the standard identity for the tangent of a sum applies when $infty$ occurs among the values of the functions involved. To address that we should also take $infty$ (not $+infty$ and not $-infty$) to be the value of a rational function wherever it has a vertical asymptote. This makes rational functions everywhere continuous. Then we have
                                  begin{align}
                                  tan(alpha+beta) & overset{hugetext{?}}= frac{sin(alpha+beta)}{cos(alpha+beta)} tag 1 \[10pt]
                                  & {} = frac{sinalphacosbeta+cosalphasinbeta}{cosalphacosbeta-sinalphasinbeta}. tag 2
                                  end{align}
                                  Notice that in $(1)$, the sine and cosine cannot both be $0$, so we need not consider what happens then.



                                  In the standard argument, we divide both the numerator and the denominator by $cosalphacosbeta$, getting this:
                                  $$
                                  frac{sinalphacosbeta+cosalphasinbeta}{cosalphacosbeta-sinalphasinbeta} = frac{frac{sinalpha}{cosalpha} - frac{sinbeta}{cosbeta}}{1 - frac{sinalpha}{cosalpha}cdotfrac{sinbeta}{cosbeta}} tag 3
                                  $$
                                  All this is valid when there are no $0$s in denominators. We need to deal with $0$s in denominators.



                                  First consider the case in which the denominator in $(1)$ is $0$ but $cosalphane0necosbeta$. Then the denominator on the right side of $(3)$ is $0$. But the numerator on the right is not $0$, since the numerator on the left is not $0$; the numerator on the left side of $(3)$ cannot be $0$ since $sin(alpha+beta) ne 0$. Hence the standard identity holds in that case.



                                  Next consider the case where $cos(alpha+beta)=0$ and $cosalpha=0$. Then



                                  $$
                                  fracpi2 = - fracpi2 = alpha+beta = fracpi2+beta = -fracpi2 + beta
                                  $$
                                  and we're done.






                                  share|cite|improve this answer























                                    up vote
                                    2
                                    down vote










                                    up vote
                                    2
                                    down vote









                                    The tangent function should be defined to take the value $infty$ at $pmpi/2$, and this $infty$ is neither $+infty$ or $-infty$, but is the $infty$ that is approached by going either in the positive direction or the negative direction. That makes the tangent function everywhere continuous, including continuity at $pmpi/2$.



                                    If one also identifies $+pi/2$ with $-pi/2$ so that the domain of the tangent function is topologically a circle one of whose points is $+pi/2=-pi/2$, then the tangent function is one-to-one. There is only one point in its domain that maps to $infty$, namely $+pi/2=-pi/2$.



                                    After that, there remains the question of whether the standard identity for the tangent of a sum applies when $infty$ occurs among the values of the functions involved. To address that we should also take $infty$ (not $+infty$ and not $-infty$) to be the value of a rational function wherever it has a vertical asymptote. This makes rational functions everywhere continuous. Then we have
                                    begin{align}
                                    tan(alpha+beta) & overset{hugetext{?}}= frac{sin(alpha+beta)}{cos(alpha+beta)} tag 1 \[10pt]
                                    & {} = frac{sinalphacosbeta+cosalphasinbeta}{cosalphacosbeta-sinalphasinbeta}. tag 2
                                    end{align}
                                    Notice that in $(1)$, the sine and cosine cannot both be $0$, so we need not consider what happens then.



                                    In the standard argument, we divide both the numerator and the denominator by $cosalphacosbeta$, getting this:
                                    $$
                                    frac{sinalphacosbeta+cosalphasinbeta}{cosalphacosbeta-sinalphasinbeta} = frac{frac{sinalpha}{cosalpha} - frac{sinbeta}{cosbeta}}{1 - frac{sinalpha}{cosalpha}cdotfrac{sinbeta}{cosbeta}} tag 3
                                    $$
                                    All this is valid when there are no $0$s in denominators. We need to deal with $0$s in denominators.



                                    First consider the case in which the denominator in $(1)$ is $0$ but $cosalphane0necosbeta$. Then the denominator on the right side of $(3)$ is $0$. But the numerator on the right is not $0$, since the numerator on the left is not $0$; the numerator on the left side of $(3)$ cannot be $0$ since $sin(alpha+beta) ne 0$. Hence the standard identity holds in that case.



                                    Next consider the case where $cos(alpha+beta)=0$ and $cosalpha=0$. Then



                                    $$
                                    fracpi2 = - fracpi2 = alpha+beta = fracpi2+beta = -fracpi2 + beta
                                    $$
                                    and we're done.






                                    share|cite|improve this answer












                                    The tangent function should be defined to take the value $infty$ at $pmpi/2$, and this $infty$ is neither $+infty$ or $-infty$, but is the $infty$ that is approached by going either in the positive direction or the negative direction. That makes the tangent function everywhere continuous, including continuity at $pmpi/2$.



                                    If one also identifies $+pi/2$ with $-pi/2$ so that the domain of the tangent function is topologically a circle one of whose points is $+pi/2=-pi/2$, then the tangent function is one-to-one. There is only one point in its domain that maps to $infty$, namely $+pi/2=-pi/2$.



                                    After that, there remains the question of whether the standard identity for the tangent of a sum applies when $infty$ occurs among the values of the functions involved. To address that we should also take $infty$ (not $+infty$ and not $-infty$) to be the value of a rational function wherever it has a vertical asymptote. This makes rational functions everywhere continuous. Then we have
                                    begin{align}
                                    tan(alpha+beta) & overset{hugetext{?}}= frac{sin(alpha+beta)}{cos(alpha+beta)} tag 1 \[10pt]
                                    & {} = frac{sinalphacosbeta+cosalphasinbeta}{cosalphacosbeta-sinalphasinbeta}. tag 2
                                    end{align}
                                    Notice that in $(1)$, the sine and cosine cannot both be $0$, so we need not consider what happens then.



                                    In the standard argument, we divide both the numerator and the denominator by $cosalphacosbeta$, getting this:
                                    $$
                                    frac{sinalphacosbeta+cosalphasinbeta}{cosalphacosbeta-sinalphasinbeta} = frac{frac{sinalpha}{cosalpha} - frac{sinbeta}{cosbeta}}{1 - frac{sinalpha}{cosalpha}cdotfrac{sinbeta}{cosbeta}} tag 3
                                    $$
                                    All this is valid when there are no $0$s in denominators. We need to deal with $0$s in denominators.



                                    First consider the case in which the denominator in $(1)$ is $0$ but $cosalphane0necosbeta$. Then the denominator on the right side of $(3)$ is $0$. But the numerator on the right is not $0$, since the numerator on the left is not $0$; the numerator on the left side of $(3)$ cannot be $0$ since $sin(alpha+beta) ne 0$. Hence the standard identity holds in that case.



                                    Next consider the case where $cos(alpha+beta)=0$ and $cosalpha=0$. Then



                                    $$
                                    fracpi2 = - fracpi2 = alpha+beta = fracpi2+beta = -fracpi2 + beta
                                    $$
                                    and we're done.







                                    share|cite|improve this answer












                                    share|cite|improve this answer



                                    share|cite|improve this answer










                                    answered Sep 19 '15 at 2:01









                                    Michael Hardy

                                    1




                                    1






















                                        up vote
                                        2
                                        down vote













                                        You can't compute $tantheta$ when $theta=pmfracpi2$, but you can compute $costheta$ instead.
                                        For $a,bin mathbb{R}$ with $abne 1$ we have
                                        begin{eqnarray}
                                        cos^2(arctan(a)+arctan(b))&=&frac{1}{1+tan^2(arctan(a)+arctan(b))}=frac{1}{1+left(frac{a+b}{1-ab}right)^2}\
                                        &=&frac{(1-ab)^2}{(1-ab)^2+(ab)^2}.
                                        end{eqnarray}
                                        It follows that
                                        $$
                                        cos^2(f(x))=lim_{ato x,bto x^{-1}}cos^2(arctan(a)+arctan(b))=lim_{ato x,bto x^{-1}}frac{(1-ab)^2}{(1-ab)^2+(ab)^2}=0,
                                        $$
                                        i.e. $f(x)=pmfracpi2$. Since $f$ is continuous on $(-infty,0)$ and on $(0,infty)$, we deduce that
                                        $$
                                        f(x)=f(-1)=2arctan(-1)=-fracpi2 quad forall x<0,
                                        $$
                                        and
                                        $$
                                        f(x)=-f(-x)=fracpi2 quad forall x>0
                                        $$





                                        Added
                                        The function $f: xmapsto f(x)=arctan(x)+arctan(x^{-1})$ defined and differentiable of $mathbb{R}setminus{0}$. For every $xne 0$ we have
                                        $$
                                        f'(x)=dfrac{1}{1+x^2}+dfrac{-x^{-2}}{1+x^{-2}}=dfrac{1}{1+x^2}-dfrac{1}{x^2+1}=0
                                        $$
                                        Therefore, $f$ is constant on each connected component of $mathbb{R}setminus{0}$.
                                        Since
                                        $$
                                        f(1)=2arctan(1)=2dfrac{pi}{4}=dfrac{pi}{2},quad f(-1)=-f(1)=-dfrac{pi}{2},
                                        $$
                                        it follows that
                                        $$
                                        arctan(x)+arctanleft(dfrac{1}{x}right)=begin{cases}-dfrac{pi}{2} &text{ if } x<0\
                                        dfrac{pi}{2} &text{ if } x>0
                                        end{cases}.
                                        $$






                                        share|cite|improve this answer



























                                          up vote
                                          2
                                          down vote













                                          You can't compute $tantheta$ when $theta=pmfracpi2$, but you can compute $costheta$ instead.
                                          For $a,bin mathbb{R}$ with $abne 1$ we have
                                          begin{eqnarray}
                                          cos^2(arctan(a)+arctan(b))&=&frac{1}{1+tan^2(arctan(a)+arctan(b))}=frac{1}{1+left(frac{a+b}{1-ab}right)^2}\
                                          &=&frac{(1-ab)^2}{(1-ab)^2+(ab)^2}.
                                          end{eqnarray}
                                          It follows that
                                          $$
                                          cos^2(f(x))=lim_{ato x,bto x^{-1}}cos^2(arctan(a)+arctan(b))=lim_{ato x,bto x^{-1}}frac{(1-ab)^2}{(1-ab)^2+(ab)^2}=0,
                                          $$
                                          i.e. $f(x)=pmfracpi2$. Since $f$ is continuous on $(-infty,0)$ and on $(0,infty)$, we deduce that
                                          $$
                                          f(x)=f(-1)=2arctan(-1)=-fracpi2 quad forall x<0,
                                          $$
                                          and
                                          $$
                                          f(x)=-f(-x)=fracpi2 quad forall x>0
                                          $$





                                          Added
                                          The function $f: xmapsto f(x)=arctan(x)+arctan(x^{-1})$ defined and differentiable of $mathbb{R}setminus{0}$. For every $xne 0$ we have
                                          $$
                                          f'(x)=dfrac{1}{1+x^2}+dfrac{-x^{-2}}{1+x^{-2}}=dfrac{1}{1+x^2}-dfrac{1}{x^2+1}=0
                                          $$
                                          Therefore, $f$ is constant on each connected component of $mathbb{R}setminus{0}$.
                                          Since
                                          $$
                                          f(1)=2arctan(1)=2dfrac{pi}{4}=dfrac{pi}{2},quad f(-1)=-f(1)=-dfrac{pi}{2},
                                          $$
                                          it follows that
                                          $$
                                          arctan(x)+arctanleft(dfrac{1}{x}right)=begin{cases}-dfrac{pi}{2} &text{ if } x<0\
                                          dfrac{pi}{2} &text{ if } x>0
                                          end{cases}.
                                          $$






                                          share|cite|improve this answer

























                                            up vote
                                            2
                                            down vote










                                            up vote
                                            2
                                            down vote









                                            You can't compute $tantheta$ when $theta=pmfracpi2$, but you can compute $costheta$ instead.
                                            For $a,bin mathbb{R}$ with $abne 1$ we have
                                            begin{eqnarray}
                                            cos^2(arctan(a)+arctan(b))&=&frac{1}{1+tan^2(arctan(a)+arctan(b))}=frac{1}{1+left(frac{a+b}{1-ab}right)^2}\
                                            &=&frac{(1-ab)^2}{(1-ab)^2+(ab)^2}.
                                            end{eqnarray}
                                            It follows that
                                            $$
                                            cos^2(f(x))=lim_{ato x,bto x^{-1}}cos^2(arctan(a)+arctan(b))=lim_{ato x,bto x^{-1}}frac{(1-ab)^2}{(1-ab)^2+(ab)^2}=0,
                                            $$
                                            i.e. $f(x)=pmfracpi2$. Since $f$ is continuous on $(-infty,0)$ and on $(0,infty)$, we deduce that
                                            $$
                                            f(x)=f(-1)=2arctan(-1)=-fracpi2 quad forall x<0,
                                            $$
                                            and
                                            $$
                                            f(x)=-f(-x)=fracpi2 quad forall x>0
                                            $$





                                            Added
                                            The function $f: xmapsto f(x)=arctan(x)+arctan(x^{-1})$ defined and differentiable of $mathbb{R}setminus{0}$. For every $xne 0$ we have
                                            $$
                                            f'(x)=dfrac{1}{1+x^2}+dfrac{-x^{-2}}{1+x^{-2}}=dfrac{1}{1+x^2}-dfrac{1}{x^2+1}=0
                                            $$
                                            Therefore, $f$ is constant on each connected component of $mathbb{R}setminus{0}$.
                                            Since
                                            $$
                                            f(1)=2arctan(1)=2dfrac{pi}{4}=dfrac{pi}{2},quad f(-1)=-f(1)=-dfrac{pi}{2},
                                            $$
                                            it follows that
                                            $$
                                            arctan(x)+arctanleft(dfrac{1}{x}right)=begin{cases}-dfrac{pi}{2} &text{ if } x<0\
                                            dfrac{pi}{2} &text{ if } x>0
                                            end{cases}.
                                            $$






                                            share|cite|improve this answer














                                            You can't compute $tantheta$ when $theta=pmfracpi2$, but you can compute $costheta$ instead.
                                            For $a,bin mathbb{R}$ with $abne 1$ we have
                                            begin{eqnarray}
                                            cos^2(arctan(a)+arctan(b))&=&frac{1}{1+tan^2(arctan(a)+arctan(b))}=frac{1}{1+left(frac{a+b}{1-ab}right)^2}\
                                            &=&frac{(1-ab)^2}{(1-ab)^2+(ab)^2}.
                                            end{eqnarray}
                                            It follows that
                                            $$
                                            cos^2(f(x))=lim_{ato x,bto x^{-1}}cos^2(arctan(a)+arctan(b))=lim_{ato x,bto x^{-1}}frac{(1-ab)^2}{(1-ab)^2+(ab)^2}=0,
                                            $$
                                            i.e. $f(x)=pmfracpi2$. Since $f$ is continuous on $(-infty,0)$ and on $(0,infty)$, we deduce that
                                            $$
                                            f(x)=f(-1)=2arctan(-1)=-fracpi2 quad forall x<0,
                                            $$
                                            and
                                            $$
                                            f(x)=-f(-x)=fracpi2 quad forall x>0
                                            $$





                                            Added
                                            The function $f: xmapsto f(x)=arctan(x)+arctan(x^{-1})$ defined and differentiable of $mathbb{R}setminus{0}$. For every $xne 0$ we have
                                            $$
                                            f'(x)=dfrac{1}{1+x^2}+dfrac{-x^{-2}}{1+x^{-2}}=dfrac{1}{1+x^2}-dfrac{1}{x^2+1}=0
                                            $$
                                            Therefore, $f$ is constant on each connected component of $mathbb{R}setminus{0}$.
                                            Since
                                            $$
                                            f(1)=2arctan(1)=2dfrac{pi}{4}=dfrac{pi}{2},quad f(-1)=-f(1)=-dfrac{pi}{2},
                                            $$
                                            it follows that
                                            $$
                                            arctan(x)+arctanleft(dfrac{1}{x}right)=begin{cases}-dfrac{pi}{2} &text{ if } x<0\
                                            dfrac{pi}{2} &text{ if } x>0
                                            end{cases}.
                                            $$







                                            share|cite|improve this answer














                                            share|cite|improve this answer



                                            share|cite|improve this answer








                                            edited Feb 17 '17 at 5:56

























                                            answered Sep 18 '15 at 21:15









                                            Mercy King

                                            13.9k11327




                                            13.9k11327






















                                                up vote
                                                1
                                                down vote













                                                I just figured I'd throw this in.



                                                Let $f(x) = arctan(x) + arctan(1/x)$ for all $x in (0, infty)$.



                                                Then $f'(x) = dfrac{1}{1+x^2} - dfrac{dfrac{1}{x^2}}{1 + dfrac{1}{x^2}} = 0$.



                                                Hence $f(x)$ is constant on $(0, infty)$.



                                                Since $f(1) = dfrac{pi}{4} + dfrac{pi}{4} = dfrac{pi}{2}$,
                                                we conclude that



                                                $f(x) = dfrac{pi}{2}$ for all $x in (0, infty)$.





                                                Addendum



                                                If you're not ready for calculus, for the same
                                                $x in (0, infty)$,
                                                Consider the point
                                                $P = (1, x)$, in the first quadrant, with corresponding angle
                                                $0 lt theta lt dfrac{pi}{2}$.



                                                Let $hat{theta} = dfrac{pi}{2} - theta$.
                                                Then, also, $0 lt hat{theta} lt dfrac{pi}{2}$ and
                                                $tan(hat theta)
                                                = tan left( dfrac{pi}{2} - theta right)
                                                = cot theta = dfrac 1x$



                                                It follows that
                                                $arctan x + arctan dfrac 1x
                                                = theta + hat theta
                                                = dfrac{pi}{2}$





                                                For all $x in (-infty, 0)$, we have



                                                $arctan x + arctan dfrac 1x =
                                                -left(arctan(-x) + arctan left(-dfrac 1x right) right) =
                                                -f(-x) = -dfrac{pi}{2}$.






                                                share|cite|improve this answer



























                                                  up vote
                                                  1
                                                  down vote













                                                  I just figured I'd throw this in.



                                                  Let $f(x) = arctan(x) + arctan(1/x)$ for all $x in (0, infty)$.



                                                  Then $f'(x) = dfrac{1}{1+x^2} - dfrac{dfrac{1}{x^2}}{1 + dfrac{1}{x^2}} = 0$.



                                                  Hence $f(x)$ is constant on $(0, infty)$.



                                                  Since $f(1) = dfrac{pi}{4} + dfrac{pi}{4} = dfrac{pi}{2}$,
                                                  we conclude that



                                                  $f(x) = dfrac{pi}{2}$ for all $x in (0, infty)$.





                                                  Addendum



                                                  If you're not ready for calculus, for the same
                                                  $x in (0, infty)$,
                                                  Consider the point
                                                  $P = (1, x)$, in the first quadrant, with corresponding angle
                                                  $0 lt theta lt dfrac{pi}{2}$.



                                                  Let $hat{theta} = dfrac{pi}{2} - theta$.
                                                  Then, also, $0 lt hat{theta} lt dfrac{pi}{2}$ and
                                                  $tan(hat theta)
                                                  = tan left( dfrac{pi}{2} - theta right)
                                                  = cot theta = dfrac 1x$



                                                  It follows that
                                                  $arctan x + arctan dfrac 1x
                                                  = theta + hat theta
                                                  = dfrac{pi}{2}$





                                                  For all $x in (-infty, 0)$, we have



                                                  $arctan x + arctan dfrac 1x =
                                                  -left(arctan(-x) + arctan left(-dfrac 1x right) right) =
                                                  -f(-x) = -dfrac{pi}{2}$.






                                                  share|cite|improve this answer

























                                                    up vote
                                                    1
                                                    down vote










                                                    up vote
                                                    1
                                                    down vote









                                                    I just figured I'd throw this in.



                                                    Let $f(x) = arctan(x) + arctan(1/x)$ for all $x in (0, infty)$.



                                                    Then $f'(x) = dfrac{1}{1+x^2} - dfrac{dfrac{1}{x^2}}{1 + dfrac{1}{x^2}} = 0$.



                                                    Hence $f(x)$ is constant on $(0, infty)$.



                                                    Since $f(1) = dfrac{pi}{4} + dfrac{pi}{4} = dfrac{pi}{2}$,
                                                    we conclude that



                                                    $f(x) = dfrac{pi}{2}$ for all $x in (0, infty)$.





                                                    Addendum



                                                    If you're not ready for calculus, for the same
                                                    $x in (0, infty)$,
                                                    Consider the point
                                                    $P = (1, x)$, in the first quadrant, with corresponding angle
                                                    $0 lt theta lt dfrac{pi}{2}$.



                                                    Let $hat{theta} = dfrac{pi}{2} - theta$.
                                                    Then, also, $0 lt hat{theta} lt dfrac{pi}{2}$ and
                                                    $tan(hat theta)
                                                    = tan left( dfrac{pi}{2} - theta right)
                                                    = cot theta = dfrac 1x$



                                                    It follows that
                                                    $arctan x + arctan dfrac 1x
                                                    = theta + hat theta
                                                    = dfrac{pi}{2}$





                                                    For all $x in (-infty, 0)$, we have



                                                    $arctan x + arctan dfrac 1x =
                                                    -left(arctan(-x) + arctan left(-dfrac 1x right) right) =
                                                    -f(-x) = -dfrac{pi}{2}$.






                                                    share|cite|improve this answer














                                                    I just figured I'd throw this in.



                                                    Let $f(x) = arctan(x) + arctan(1/x)$ for all $x in (0, infty)$.



                                                    Then $f'(x) = dfrac{1}{1+x^2} - dfrac{dfrac{1}{x^2}}{1 + dfrac{1}{x^2}} = 0$.



                                                    Hence $f(x)$ is constant on $(0, infty)$.



                                                    Since $f(1) = dfrac{pi}{4} + dfrac{pi}{4} = dfrac{pi}{2}$,
                                                    we conclude that



                                                    $f(x) = dfrac{pi}{2}$ for all $x in (0, infty)$.





                                                    Addendum



                                                    If you're not ready for calculus, for the same
                                                    $x in (0, infty)$,
                                                    Consider the point
                                                    $P = (1, x)$, in the first quadrant, with corresponding angle
                                                    $0 lt theta lt dfrac{pi}{2}$.



                                                    Let $hat{theta} = dfrac{pi}{2} - theta$.
                                                    Then, also, $0 lt hat{theta} lt dfrac{pi}{2}$ and
                                                    $tan(hat theta)
                                                    = tan left( dfrac{pi}{2} - theta right)
                                                    = cot theta = dfrac 1x$



                                                    It follows that
                                                    $arctan x + arctan dfrac 1x
                                                    = theta + hat theta
                                                    = dfrac{pi}{2}$





                                                    For all $x in (-infty, 0)$, we have



                                                    $arctan x + arctan dfrac 1x =
                                                    -left(arctan(-x) + arctan left(-dfrac 1x right) right) =
                                                    -f(-x) = -dfrac{pi}{2}$.







                                                    share|cite|improve this answer














                                                    share|cite|improve this answer



                                                    share|cite|improve this answer








                                                    edited Sep 19 '15 at 1:49

























                                                    answered Sep 19 '15 at 0:55









                                                    steven gregory

                                                    17.6k22257




                                                    17.6k22257






















                                                        up vote
                                                        0
                                                        down vote













                                                        Your logic is entirely correct.



                                                        The functions $tan^{-1} x $ and $tan^{-1} (1/x ) $ are both odd functions. Accordingly their additive $tan^{-1} x + tan^{-1} (1/x ) $ must be odd with $ + pi/2 $ value for positive arguments and $ - pi/2 $ value for negative arguments.



                                                        This function results by subtracting $ pi/2$ from the step function 0 to $pi$ jump at x = 0 and so making it an odd function of opposite sign for arguments of opposite sign, defining the $ f(x) $ to be $pi/2 $ for $ x>0, 0$ for $ x=0 $ and $ - pi/2 $ for $ x<0. $






                                                        share|cite|improve this answer



























                                                          up vote
                                                          0
                                                          down vote













                                                          Your logic is entirely correct.



                                                          The functions $tan^{-1} x $ and $tan^{-1} (1/x ) $ are both odd functions. Accordingly their additive $tan^{-1} x + tan^{-1} (1/x ) $ must be odd with $ + pi/2 $ value for positive arguments and $ - pi/2 $ value for negative arguments.



                                                          This function results by subtracting $ pi/2$ from the step function 0 to $pi$ jump at x = 0 and so making it an odd function of opposite sign for arguments of opposite sign, defining the $ f(x) $ to be $pi/2 $ for $ x>0, 0$ for $ x=0 $ and $ - pi/2 $ for $ x<0. $






                                                          share|cite|improve this answer

























                                                            up vote
                                                            0
                                                            down vote










                                                            up vote
                                                            0
                                                            down vote









                                                            Your logic is entirely correct.



                                                            The functions $tan^{-1} x $ and $tan^{-1} (1/x ) $ are both odd functions. Accordingly their additive $tan^{-1} x + tan^{-1} (1/x ) $ must be odd with $ + pi/2 $ value for positive arguments and $ - pi/2 $ value for negative arguments.



                                                            This function results by subtracting $ pi/2$ from the step function 0 to $pi$ jump at x = 0 and so making it an odd function of opposite sign for arguments of opposite sign, defining the $ f(x) $ to be $pi/2 $ for $ x>0, 0$ for $ x=0 $ and $ - pi/2 $ for $ x<0. $






                                                            share|cite|improve this answer














                                                            Your logic is entirely correct.



                                                            The functions $tan^{-1} x $ and $tan^{-1} (1/x ) $ are both odd functions. Accordingly their additive $tan^{-1} x + tan^{-1} (1/x ) $ must be odd with $ + pi/2 $ value for positive arguments and $ - pi/2 $ value for negative arguments.



                                                            This function results by subtracting $ pi/2$ from the step function 0 to $pi$ jump at x = 0 and so making it an odd function of opposite sign for arguments of opposite sign, defining the $ f(x) $ to be $pi/2 $ for $ x>0, 0$ for $ x=0 $ and $ - pi/2 $ for $ x<0. $







                                                            share|cite|improve this answer














                                                            share|cite|improve this answer



                                                            share|cite|improve this answer








                                                            edited Feb 16 '17 at 21:37

























                                                            answered Sep 19 '15 at 2:48









                                                            Narasimham

                                                            20.4k52158




                                                            20.4k52158






















                                                                up vote
                                                                0
                                                                down vote













                                                                An alternative proof using calculus.



                                                                Let $f(x) = arctan(x) + arctan(1/x)$ for all real $x$ except $0$.



                                                                $$frac {df(x)} {dx}= frac{1}{x^2+1} + frac {1}{1+1/x^2}*frac{-1}{x^2} = frac{1}{x^2+1} - frac{1}{x^2+1} = 0 $$



                                                                Integrating on both sides we get



                                                                $$f(x) = arctan(x) + arctan(1/x) = C,$$ where $C$ is a constant.



                                                                Since the function is discontinuous at $x=0$ the constant value can be different on both sides.



                                                                It can be obtained for $x>0$ by for example setting $x=1$:
                                                                $$f(1) = arctan(1) + arctan(1) = pi/4 + pi/4 = pi/2 $$
                                                                Similarly, for $x=-1$:
                                                                $$f(1) = arctan(-1) + arctan(-1) = -pi/4 -pi/4 = -pi/2 $$



                                                                So, in conclusion, $$arctan(x) + arctan(1/x) = pi/2 $$ for $x>0$
                                                                and
                                                                $$arctan(x) + arctan(1/x) = -pi/2 $$ for $x<0$.






                                                                share|cite|improve this answer



























                                                                  up vote
                                                                  0
                                                                  down vote













                                                                  An alternative proof using calculus.



                                                                  Let $f(x) = arctan(x) + arctan(1/x)$ for all real $x$ except $0$.



                                                                  $$frac {df(x)} {dx}= frac{1}{x^2+1} + frac {1}{1+1/x^2}*frac{-1}{x^2} = frac{1}{x^2+1} - frac{1}{x^2+1} = 0 $$



                                                                  Integrating on both sides we get



                                                                  $$f(x) = arctan(x) + arctan(1/x) = C,$$ where $C$ is a constant.



                                                                  Since the function is discontinuous at $x=0$ the constant value can be different on both sides.



                                                                  It can be obtained for $x>0$ by for example setting $x=1$:
                                                                  $$f(1) = arctan(1) + arctan(1) = pi/4 + pi/4 = pi/2 $$
                                                                  Similarly, for $x=-1$:
                                                                  $$f(1) = arctan(-1) + arctan(-1) = -pi/4 -pi/4 = -pi/2 $$



                                                                  So, in conclusion, $$arctan(x) + arctan(1/x) = pi/2 $$ for $x>0$
                                                                  and
                                                                  $$arctan(x) + arctan(1/x) = -pi/2 $$ for $x<0$.






                                                                  share|cite|improve this answer

























                                                                    up vote
                                                                    0
                                                                    down vote










                                                                    up vote
                                                                    0
                                                                    down vote









                                                                    An alternative proof using calculus.



                                                                    Let $f(x) = arctan(x) + arctan(1/x)$ for all real $x$ except $0$.



                                                                    $$frac {df(x)} {dx}= frac{1}{x^2+1} + frac {1}{1+1/x^2}*frac{-1}{x^2} = frac{1}{x^2+1} - frac{1}{x^2+1} = 0 $$



                                                                    Integrating on both sides we get



                                                                    $$f(x) = arctan(x) + arctan(1/x) = C,$$ where $C$ is a constant.



                                                                    Since the function is discontinuous at $x=0$ the constant value can be different on both sides.



                                                                    It can be obtained for $x>0$ by for example setting $x=1$:
                                                                    $$f(1) = arctan(1) + arctan(1) = pi/4 + pi/4 = pi/2 $$
                                                                    Similarly, for $x=-1$:
                                                                    $$f(1) = arctan(-1) + arctan(-1) = -pi/4 -pi/4 = -pi/2 $$



                                                                    So, in conclusion, $$arctan(x) + arctan(1/x) = pi/2 $$ for $x>0$
                                                                    and
                                                                    $$arctan(x) + arctan(1/x) = -pi/2 $$ for $x<0$.






                                                                    share|cite|improve this answer














                                                                    An alternative proof using calculus.



                                                                    Let $f(x) = arctan(x) + arctan(1/x)$ for all real $x$ except $0$.



                                                                    $$frac {df(x)} {dx}= frac{1}{x^2+1} + frac {1}{1+1/x^2}*frac{-1}{x^2} = frac{1}{x^2+1} - frac{1}{x^2+1} = 0 $$



                                                                    Integrating on both sides we get



                                                                    $$f(x) = arctan(x) + arctan(1/x) = C,$$ where $C$ is a constant.



                                                                    Since the function is discontinuous at $x=0$ the constant value can be different on both sides.



                                                                    It can be obtained for $x>0$ by for example setting $x=1$:
                                                                    $$f(1) = arctan(1) + arctan(1) = pi/4 + pi/4 = pi/2 $$
                                                                    Similarly, for $x=-1$:
                                                                    $$f(1) = arctan(-1) + arctan(-1) = -pi/4 -pi/4 = -pi/2 $$



                                                                    So, in conclusion, $$arctan(x) + arctan(1/x) = pi/2 $$ for $x>0$
                                                                    and
                                                                    $$arctan(x) + arctan(1/x) = -pi/2 $$ for $x<0$.







                                                                    share|cite|improve this answer














                                                                    share|cite|improve this answer



                                                                    share|cite|improve this answer








                                                                    edited Nov 14 at 21:36









                                                                    Robert Howard

                                                                    1,742722




                                                                    1,742722










                                                                    answered Nov 14 at 21:06









                                                                    Adrian

                                                                    11




                                                                    11






























                                                                         

                                                                        draft saved


                                                                        draft discarded



















































                                                                         


                                                                        draft saved


                                                                        draft discarded














                                                                        StackExchange.ready(
                                                                        function () {
                                                                        StackExchange.openid.initPostLogin('.new-post-login', 'https%3a%2f%2fmath.stackexchange.com%2fquestions%2f1430520%2fproving-that-arctanx-arctan1-x-pm-pi-2-could-this-line-of-reasoning-p%23new-answer', 'question_page');
                                                                        }
                                                                        );

                                                                        Post as a guest















                                                                        Required, but never shown





















































                                                                        Required, but never shown














                                                                        Required, but never shown












                                                                        Required, but never shown







                                                                        Required, but never shown

































                                                                        Required, but never shown














                                                                        Required, but never shown












                                                                        Required, but never shown







                                                                        Required, but never shown







                                                                        Popular posts from this blog

                                                                        Biblatex bibliography style without URLs when DOI exists (in Overleaf with Zotero bibliography)

                                                                        ComboBox Display Member on multiple fields

                                                                        Is it possible to collect Nectar points via Trainline?